Inverse of a function containing $(-1)^n$












1












$begingroup$


Is it possible to find the inverse of a function which contains the term $(-1)^n$?



Specifically: I want to find the inverse of the following function, which gives the nth number of the form $6kpm1$:
$$ f(n) = 3n + frac{3}{2} - frac{(-1)^n}{2} $$



Since it is not possible to take the log to base -1 and I'm not a mathematician, I didn't suceed with my attempts of solving for $f(n)$. (Google search and Wolfram Alpha didn't help either, and based on the fact that $(-1)^n$ "looses" information about n I doubt it's possible in general)










share|cite|improve this question











$endgroup$












  • $begingroup$
    I think you mean to tabulate numbers of the form $6kpm 1$ but your function does not do that.
    $endgroup$
    – Ross Millikan
    Jan 2 at 14:44
















1












$begingroup$


Is it possible to find the inverse of a function which contains the term $(-1)^n$?



Specifically: I want to find the inverse of the following function, which gives the nth number of the form $6kpm1$:
$$ f(n) = 3n + frac{3}{2} - frac{(-1)^n}{2} $$



Since it is not possible to take the log to base -1 and I'm not a mathematician, I didn't suceed with my attempts of solving for $f(n)$. (Google search and Wolfram Alpha didn't help either, and based on the fact that $(-1)^n$ "looses" information about n I doubt it's possible in general)










share|cite|improve this question











$endgroup$












  • $begingroup$
    I think you mean to tabulate numbers of the form $6kpm 1$ but your function does not do that.
    $endgroup$
    – Ross Millikan
    Jan 2 at 14:44














1












1








1





$begingroup$


Is it possible to find the inverse of a function which contains the term $(-1)^n$?



Specifically: I want to find the inverse of the following function, which gives the nth number of the form $6kpm1$:
$$ f(n) = 3n + frac{3}{2} - frac{(-1)^n}{2} $$



Since it is not possible to take the log to base -1 and I'm not a mathematician, I didn't suceed with my attempts of solving for $f(n)$. (Google search and Wolfram Alpha didn't help either, and based on the fact that $(-1)^n$ "looses" information about n I doubt it's possible in general)










share|cite|improve this question











$endgroup$




Is it possible to find the inverse of a function which contains the term $(-1)^n$?



Specifically: I want to find the inverse of the following function, which gives the nth number of the form $6kpm1$:
$$ f(n) = 3n + frac{3}{2} - frac{(-1)^n}{2} $$



Since it is not possible to take the log to base -1 and I'm not a mathematician, I didn't suceed with my attempts of solving for $f(n)$. (Google search and Wolfram Alpha didn't help either, and based on the fact that $(-1)^n$ "looses" information about n I doubt it's possible in general)







inverse-function






share|cite|improve this question















share|cite|improve this question













share|cite|improve this question




share|cite|improve this question








edited Jan 2 at 15:22







g3nuine

















asked Jan 2 at 14:38









g3nuineg3nuine

1598




1598












  • $begingroup$
    I think you mean to tabulate numbers of the form $6kpm 1$ but your function does not do that.
    $endgroup$
    – Ross Millikan
    Jan 2 at 14:44


















  • $begingroup$
    I think you mean to tabulate numbers of the form $6kpm 1$ but your function does not do that.
    $endgroup$
    – Ross Millikan
    Jan 2 at 14:44
















$begingroup$
I think you mean to tabulate numbers of the form $6kpm 1$ but your function does not do that.
$endgroup$
– Ross Millikan
Jan 2 at 14:44




$begingroup$
I think you mean to tabulate numbers of the form $6kpm 1$ but your function does not do that.
$endgroup$
– Ross Millikan
Jan 2 at 14:44










2 Answers
2






active

oldest

votes


















3












$begingroup$

For $n$ even we have $$f(n)=3n+1$$ and you can write $$frac{f(n)-1}{3}=n$$
If $n$ is odd then you will get $$f(n)=3n+2$$ so $$n=frac{f(n)-2}{3}$$






share|cite|improve this answer









$endgroup$













  • $begingroup$
    The problem is that I don't know n's parity im advance, but I guess a single function being the inverse instead of two is not possible?
    $endgroup$
    – g3nuine
    Jan 2 at 15:28










  • $begingroup$
    I think only one term as inverse is not possible.
    $endgroup$
    – Dr. Sonnhard Graubner
    Jan 2 at 15:30



















1












$begingroup$

Consider the equation $f(n)=m$:




  • if $n=2k$, you get the equation $6k+1=m$, whose solution is $k=frac{m-1}6left(iff n=frac{m-1}3right)$;

  • if $n=2k-1$, you get the equation $6k-1=m$, whose solution is $k=frac{m+1}6left(iff n=frac{m+1}3right)$.


So, the inverse that you're looking for is$$nmapstofrac{n-(-1)^n}3.$$






share|cite|improve this answer









$endgroup$














    Your Answer





    StackExchange.ifUsing("editor", function () {
    return StackExchange.using("mathjaxEditing", function () {
    StackExchange.MarkdownEditor.creationCallbacks.add(function (editor, postfix) {
    StackExchange.mathjaxEditing.prepareWmdForMathJax(editor, postfix, [["$", "$"], ["\\(","\\)"]]);
    });
    });
    }, "mathjax-editing");

    StackExchange.ready(function() {
    var channelOptions = {
    tags: "".split(" "),
    id: "69"
    };
    initTagRenderer("".split(" "), "".split(" "), channelOptions);

    StackExchange.using("externalEditor", function() {
    // Have to fire editor after snippets, if snippets enabled
    if (StackExchange.settings.snippets.snippetsEnabled) {
    StackExchange.using("snippets", function() {
    createEditor();
    });
    }
    else {
    createEditor();
    }
    });

    function createEditor() {
    StackExchange.prepareEditor({
    heartbeatType: 'answer',
    autoActivateHeartbeat: false,
    convertImagesToLinks: true,
    noModals: true,
    showLowRepImageUploadWarning: true,
    reputationToPostImages: 10,
    bindNavPrevention: true,
    postfix: "",
    imageUploader: {
    brandingHtml: "Powered by u003ca class="icon-imgur-white" href="https://imgur.com/"u003eu003c/au003e",
    contentPolicyHtml: "User contributions licensed under u003ca href="https://creativecommons.org/licenses/by-sa/3.0/"u003ecc by-sa 3.0 with attribution requiredu003c/au003e u003ca href="https://stackoverflow.com/legal/content-policy"u003e(content policy)u003c/au003e",
    allowUrls: true
    },
    noCode: true, onDemand: true,
    discardSelector: ".discard-answer"
    ,immediatelyShowMarkdownHelp:true
    });


    }
    });














    draft saved

    draft discarded


















    StackExchange.ready(
    function () {
    StackExchange.openid.initPostLogin('.new-post-login', 'https%3a%2f%2fmath.stackexchange.com%2fquestions%2f3059536%2finverse-of-a-function-containing-1n%23new-answer', 'question_page');
    }
    );

    Post as a guest















    Required, but never shown

























    2 Answers
    2






    active

    oldest

    votes








    2 Answers
    2






    active

    oldest

    votes









    active

    oldest

    votes






    active

    oldest

    votes









    3












    $begingroup$

    For $n$ even we have $$f(n)=3n+1$$ and you can write $$frac{f(n)-1}{3}=n$$
    If $n$ is odd then you will get $$f(n)=3n+2$$ so $$n=frac{f(n)-2}{3}$$






    share|cite|improve this answer









    $endgroup$













    • $begingroup$
      The problem is that I don't know n's parity im advance, but I guess a single function being the inverse instead of two is not possible?
      $endgroup$
      – g3nuine
      Jan 2 at 15:28










    • $begingroup$
      I think only one term as inverse is not possible.
      $endgroup$
      – Dr. Sonnhard Graubner
      Jan 2 at 15:30
















    3












    $begingroup$

    For $n$ even we have $$f(n)=3n+1$$ and you can write $$frac{f(n)-1}{3}=n$$
    If $n$ is odd then you will get $$f(n)=3n+2$$ so $$n=frac{f(n)-2}{3}$$






    share|cite|improve this answer









    $endgroup$













    • $begingroup$
      The problem is that I don't know n's parity im advance, but I guess a single function being the inverse instead of two is not possible?
      $endgroup$
      – g3nuine
      Jan 2 at 15:28










    • $begingroup$
      I think only one term as inverse is not possible.
      $endgroup$
      – Dr. Sonnhard Graubner
      Jan 2 at 15:30














    3












    3








    3





    $begingroup$

    For $n$ even we have $$f(n)=3n+1$$ and you can write $$frac{f(n)-1}{3}=n$$
    If $n$ is odd then you will get $$f(n)=3n+2$$ so $$n=frac{f(n)-2}{3}$$






    share|cite|improve this answer









    $endgroup$



    For $n$ even we have $$f(n)=3n+1$$ and you can write $$frac{f(n)-1}{3}=n$$
    If $n$ is odd then you will get $$f(n)=3n+2$$ so $$n=frac{f(n)-2}{3}$$







    share|cite|improve this answer












    share|cite|improve this answer



    share|cite|improve this answer










    answered Jan 2 at 14:46









    Dr. Sonnhard GraubnerDr. Sonnhard Graubner

    78.4k42867




    78.4k42867












    • $begingroup$
      The problem is that I don't know n's parity im advance, but I guess a single function being the inverse instead of two is not possible?
      $endgroup$
      – g3nuine
      Jan 2 at 15:28










    • $begingroup$
      I think only one term as inverse is not possible.
      $endgroup$
      – Dr. Sonnhard Graubner
      Jan 2 at 15:30


















    • $begingroup$
      The problem is that I don't know n's parity im advance, but I guess a single function being the inverse instead of two is not possible?
      $endgroup$
      – g3nuine
      Jan 2 at 15:28










    • $begingroup$
      I think only one term as inverse is not possible.
      $endgroup$
      – Dr. Sonnhard Graubner
      Jan 2 at 15:30
















    $begingroup$
    The problem is that I don't know n's parity im advance, but I guess a single function being the inverse instead of two is not possible?
    $endgroup$
    – g3nuine
    Jan 2 at 15:28




    $begingroup$
    The problem is that I don't know n's parity im advance, but I guess a single function being the inverse instead of two is not possible?
    $endgroup$
    – g3nuine
    Jan 2 at 15:28












    $begingroup$
    I think only one term as inverse is not possible.
    $endgroup$
    – Dr. Sonnhard Graubner
    Jan 2 at 15:30




    $begingroup$
    I think only one term as inverse is not possible.
    $endgroup$
    – Dr. Sonnhard Graubner
    Jan 2 at 15:30











    1












    $begingroup$

    Consider the equation $f(n)=m$:




    • if $n=2k$, you get the equation $6k+1=m$, whose solution is $k=frac{m-1}6left(iff n=frac{m-1}3right)$;

    • if $n=2k-1$, you get the equation $6k-1=m$, whose solution is $k=frac{m+1}6left(iff n=frac{m+1}3right)$.


    So, the inverse that you're looking for is$$nmapstofrac{n-(-1)^n}3.$$






    share|cite|improve this answer









    $endgroup$


















      1












      $begingroup$

      Consider the equation $f(n)=m$:




      • if $n=2k$, you get the equation $6k+1=m$, whose solution is $k=frac{m-1}6left(iff n=frac{m-1}3right)$;

      • if $n=2k-1$, you get the equation $6k-1=m$, whose solution is $k=frac{m+1}6left(iff n=frac{m+1}3right)$.


      So, the inverse that you're looking for is$$nmapstofrac{n-(-1)^n}3.$$






      share|cite|improve this answer









      $endgroup$
















        1












        1








        1





        $begingroup$

        Consider the equation $f(n)=m$:




        • if $n=2k$, you get the equation $6k+1=m$, whose solution is $k=frac{m-1}6left(iff n=frac{m-1}3right)$;

        • if $n=2k-1$, you get the equation $6k-1=m$, whose solution is $k=frac{m+1}6left(iff n=frac{m+1}3right)$.


        So, the inverse that you're looking for is$$nmapstofrac{n-(-1)^n}3.$$






        share|cite|improve this answer









        $endgroup$



        Consider the equation $f(n)=m$:




        • if $n=2k$, you get the equation $6k+1=m$, whose solution is $k=frac{m-1}6left(iff n=frac{m-1}3right)$;

        • if $n=2k-1$, you get the equation $6k-1=m$, whose solution is $k=frac{m+1}6left(iff n=frac{m+1}3right)$.


        So, the inverse that you're looking for is$$nmapstofrac{n-(-1)^n}3.$$







        share|cite|improve this answer












        share|cite|improve this answer



        share|cite|improve this answer










        answered Jan 2 at 14:45









        José Carlos SantosJosé Carlos Santos

        171k23132240




        171k23132240






























            draft saved

            draft discarded




















































            Thanks for contributing an answer to Mathematics Stack Exchange!


            • Please be sure to answer the question. Provide details and share your research!

            But avoid



            • Asking for help, clarification, or responding to other answers.

            • Making statements based on opinion; back them up with references or personal experience.


            Use MathJax to format equations. MathJax reference.


            To learn more, see our tips on writing great answers.




            draft saved


            draft discarded














            StackExchange.ready(
            function () {
            StackExchange.openid.initPostLogin('.new-post-login', 'https%3a%2f%2fmath.stackexchange.com%2fquestions%2f3059536%2finverse-of-a-function-containing-1n%23new-answer', 'question_page');
            }
            );

            Post as a guest















            Required, but never shown





















































            Required, but never shown














            Required, but never shown












            Required, but never shown







            Required, but never shown

































            Required, but never shown














            Required, but never shown












            Required, but never shown







            Required, but never shown







            Popular posts from this blog

            How do I know what Microsoft account the skydrive app is syncing to?

            When does type information flow backwards in C++?

            Grease: Live!